Lesson: Flawed Argument Questions

Comment on Flawed Argument Questions

Could you please help in understanding how the answer to the 650-800 question is "D" ?

The violent crime rate (number of violent crimes per 1,000 residents) in Meadowbrook is 60 percent higher now than it was four years ago. The corresponding increase for Parkdale is only 10 percent. These figures support the conclusion that residents of Meadowbrook are more likely to become victims of violent crime than are residents of Parkdale.

The argument above is flawed because it fails to take into account
A. changes in the population density of both Parkdale and Meadowbrook over the past four years
B. how the rate of population growth in Meadowbrook over the past four years compares to the corresponding rate for Parkdale
C. the ratio of violent to nonviolent crimes committed during the past four years in Meadowbrook and Parkdale
D. the violent crime rates in Meadowbrook and Parkdale four years ago
E. how Meadowbrook’s expenditures for crime prevention over the past four years compare to Parkdale’s expenditures
gmat-admin's picture

You're referring to https://gmatclub.com/forum/the-violent-crime-rate-number-of-violent-crim...

The passage SUGGESTS that the violent crime rate in Meadowbrook than the violent crime rate in Parkdale.

Although Meadowbrook had a greater 4-year INCREASE (60%) than the increase that Parkdale had (10%), we don't know anything about the actual crime RATES in these cities.

Consider this scenario:

Meadowbrook: 4 YEARS AGO, Meadowbrook had 10 violent crimes per 1000 residents. Since the 4-year increase is 60%, the PRESENT rate is 16 violent crimes per 1000 residents.

Parkdale: 4 YEARS AGO, Parkdale had 500 violent crimes per 1000 residents. Since the 4-year increase is 10%, the PRESENT rate is 550 violent crimes per 1000 residents.

In this scenario, we can see that, although Parkdale's 4-year increase is LESS THAN Meadowbrook's 4-year increase, it is clear that Parkdale's residents are more likely to become victims of violent crime than are residents of Meadowbrook.

So, the 4-year INCREASES do not provide enough information to make any conclusions about which city's residents are more likely to become victims of violent crime. We need information about the actual crime RATES, not the 4-year increases.

Brent,

Could you explain me why B is wrong?

Thanks!
gmat-admin's picture

B is irrelevant. We are given information that compares today's crime rate with the crime rate 4 years ago. It doesn't matter what happened in between those 4 years.

Brent,

I have a real big problem. English for me is a foreign language, so I loose a lot of time to write the premises and conclusion, or to find the answer before check all the answer choices.
Do you have another strategy that could help me?

Thanks
gmat-admin's picture

This is a common problem among test-takers for whom English is a second language. You might try seeing how you do WITHOUT taking notes.

Alternatively, you may need to spend some time experimenting with different strategies to see which one best suits your reading speed and English proficiency.

For example, you may need to look for questions in the Verbal section where you immediately guess and then use that extra time to ensure a higher level of accuracy.

For example, some CR questions have incredibly long passages and very long answer choices. This might be an excellent candidate for guessing.

Keep in mind that this is a Plan B strategy. Of course, it's best to answer every question. I'm just talking about situations in which it's really hard for students to complete the Verbal section in 75 minutes.

Hi Brent,

is this question of Flawed argument?

https://gmatclub.com/forum/which-of-the-following-most-logically-completes-the-argument-75869.html
gmat-admin's picture

Great question!
Link: https://gmatclub.com/forum/which-of-the-following-most-logically-complet...

The question is part Flawed Argument and part Paradox, but I think it's closer to a Flawed Argument question.

The idea here is that, IF we were to conclude that salt causes high blood pressure, then that would be a flawed argument, BECAUSE _______

The missing part is a premise that highlights why that conclusion is flawed

https://gmatclub.com/forum/although-the-earliest-surviving-greek-inscriptions-written-in-an-alpha-82727.html
sir please provide explanation for this
gmat-admin's picture

https://gmatclub.com/forum/according-to-a-prediction-of-the-not-so-distant-future-published-in-136974.html

Hi Brent,

In this question, I went to answer C as the information mentioned by option D is already mentioned for in the question statement.

Thanks,
Erik
gmat-admin's picture

Question link: https://gmatclub.com/forum/according-to-a-prediction-of-the-not-so-dista...

Yes, answer choice D has some similarities with what's mentioned in the passage, but it still explains the key flaw (that the electricity will harm BOTH weeds and crops plants. That's all that matters.

Hi Brent,

Could you please help explain this question

https://gmatclub.com/forum/many-economists-hold-that-keeping-taxes-low-helps-to-spur-economic-gro-321891.html

Thanks

Got it... Thanks for the comparison.

Office Hours

On December 20, 2023, Brent will stop offering office hours. 

Change Playback Speed

You have the option of watching the videos at various speeds (25% faster, 50% faster, etc). To change the playback speed, click the settings icon on the right side of the video status bar.

Have a question about this video?

Post your question in the Comment section below, and a GMAT expert will answer it as fast as humanly possible.

Free “Question of the Day” emails!